K
Khách

Hãy nhập câu hỏi của bạn vào đây, nếu là tài khoản VIP, bạn sẽ được ưu tiên trả lời.

3 tháng 1 2019

Qúa dễ luôn 

Ta có : a x 2 + b x 2 + c x 2 \(\le\) 5 

           2 x ( a + b + c )        \(\le\)5

               a + b + c              \(\le\) 5/2 

               a + b + c              \(\le\) 2,5 

Mà theo đề bài : a + b + c không lớn hơn 2 ( có nghĩa là bé hơn 2 ) . Nên a + b + c phải luôn luôn bé hơn 2,5 ( vì 2 luôn bé hơn 2,5 ) 

Vậy : a x 2 + b x 2 + c x 2 \(\le\) 5 

NV
2 tháng 3 2023

Do \(0\le a;b;c\le2\) 

\(\Rightarrow abc+\left(2-a\right)\left(2-b\right)\left(2-c\right)\ge0\)

\(\Leftrightarrow2\left(ab+bc+ca\right)-4\left(a+b+c\right)+8\ge0\)

\(\Leftrightarrow2\left(ab+bc+ca\right)\ge4\)

\(\Leftrightarrow\left(a+b+c\right)^2-\left(a^2+b^2+c^2\right)\ge4\)

\(\Leftrightarrow9-\left(a^2+b^2+c^2\right)\ge4\)

\(\Leftrightarrow a^2+b^2+c^2\le5\)

Dấu "=" xảy ra khi \(\left(a;b;c\right)=\left(0;1;2\right)\) và các hoán vị

1

Áp dụng BĐT Cauchy cho 2 số dương:

4ac=2.b.2c≤2(b+2c2)2≤2(a+b+2c2)2=2.(12)2=12

⇒−4bc≥−12

⇒K=ab+4ac−4bc≥−4bc≥−12

6 tháng 3 2016
Hi a,b,c không âm và lớn hơn 2 thì sao mà a+b+c =3đc nhỉ??????
12 tháng 2 2022

Giúp mình bài này với ah.

AH
Akai Haruma
Giáo viên
12 tháng 2 2022

Lời giải:

Tìm min:

Áp dụng BĐT AM-GM:

$a^3+a^3+1\geq 3a^2$

$b^3+b^3+1\geq 3b^2$

$c^3+c^3+1\geq 3c^2$

$\Rightarrow 2(a^3+b^3+c^3)+3\geq 3(a^2+b^2+c^2)$

$\Leftrightarrow 2P+3\geq 9$

$\Leftrightarrow P\geq 3$

Vậy $P_{\min}=3$ khi $(a,b,c)=(1,1,1)$

----------------

Tìm max:

$a^2+b^2+c^2=3\Rightarrow a^2,b^2,c^2\leq 3$

$\Rightarrow a,b,c\leq \sqrt{3}$

Do đó: $a^3-\sqrt{3}a^2=a^2(a-\sqrt{3})\leq 0$

$\Rightarrow a^3\leq \sqrt{3}a^2$

Tương tự với $b,c$ và cộng theo vế:

$P\leq \sqrt{3}(a^2+b^2+c^2)=3\sqrt{3}$
Vậy $P_{\max}=3\sqrt{3}$ khi $(a,b,c)=(\sqrt{3},0,0)$ và hoán vị. 

2 tháng 5 2021

132-79=

2 tháng 5 2021

ta có :

\(\frac{a^3+b^3}{a^2+ab+b^2}=\frac{2a^3}{a^2+ab+b^2}+\frac{b^3-a^3}{a^2+ab+b^2}=\frac{2a^3}{a^2+ab+b^3}+b-a\)

tương tự rồi cộng theo vế : 

\(LHS\ge2\left(\frac{a^3}{a^2+ab+b^2}+\frac{b^3}{b^2+bc+c^2}+\frac{c^3}{c^2+ca+a^2}\right)\)

áp dụng bđt cô si

 \(\frac{a^3}{a^2+ab+b^2}+\frac{a^2+ab+b^2}{9}+\frac{1}{3}\ge\frac{3a}{3}=a\)

tương tự rồi cộng theo vế 

\(2\left(\frac{a^3}{a^2+ab+b^2}+...\right)\ge a+b+c-1-\frac{2\left(a^2+b^2+c^2+ab+bc+ca\right)}{9}\)

\(\ge\frac{2\left(9-a^2-b^2-c^2-ab-bc-ca\right)}{9}\)

đến đây chịu :)))))